Lagrangian Formalism – Derivation of Euler-Lagrange Equations for Lagrangian with Dependence on Second Order Derivatives

actionboundary conditionslagrangian-formalismvariational-calculusvariational-principle

Suppose we have a Lagrangian that depends on second-order derivatives:

$$L = L(q, \dot{q}, \ddot{q},t).\tag{1}$$

If we're working on the variational problem for this Lagrangian, then I know that we'll wind up with the following Euler-Lagrange equation:
$$\frac{\partial L}{\partial q} – \frac{d}{dt} \frac{\partial L}{\partial \dot{q}} + \frac{d^2}{dt^2} \frac{\partial L}{\partial \ddot{q}} = 0.\tag{2}$$
However, I can't see how to derive this equation. Obviously, the final term is supposed to come from integrating by parts the contribution of the $\ddot{q}$ dependence to the variation in the Lagrangian; doing that yields (writing $T$ for the period of time over which we're extremising the action):
$$\int_T \frac{\partial L}{\partial \ddot{q}} \delta \ddot{q} = \left[ \frac{\partial L}{\partial \ddot{q}} \delta \dot{q} – \frac{\partial L}{\partial \ddot{q}} \delta q \right]_{\partial T} + \int_T \frac{d^2}{dt^2} \frac{\partial L}{\partial \ddot{q}}.\tag{3}$$
Now, the term in square brackets presumably has to vanish. The right-hand term will do so, since $\delta q$ vanishes on the boundary of $T$; but why should the left-hand term vanish? Is it just a condition of dealing with the variations for such a Lagrangian that we consider only variations for which $\delta \dot{q}$ vanishes on the boundary of integration as well? Or is there something I'm missing?

Best Answer

Suppose now the Lagrangian $L$ is a function of $y(x), y'(x)$ and also $y''(x)$, i.e. it contains second derivatives w/r to the parameter $x$.

It is straightforward to adapt the usual procedure to this case: write \begin{align} Y(x,\epsilon)=y(x)+\epsilon\,\eta(x) \end{align} for an otherwise arbitrary function $\eta$.

We then have the parametrized integral \begin{align} I(\epsilon)=\displaystyle \int_a^b\,dx\,L(x,Y(x,\epsilon),Y'(x,\epsilon), Y''(x,\epsilon)) \end{align} and we want to find $L$ at $\epsilon=0$ so that \begin{align} 0&=\frac{dI}{d\epsilon}\vert_{\epsilon=0}\, ,\\ &=\displaystyle\int_a^b\,dx\, \left( \frac{\partial L}{\partial Y} \frac{\partial Y}{\partial \epsilon}+ \frac{\partial L}{\partial Y'}\frac{\partial Y'}{\partial \epsilon} +\frac{\partial L}{\partial Y''}\frac{\partial Y''}{\partial \epsilon} \right)\vert_{\epsilon=0}\, ,\\ &=\displaystyle\int_a^b\,dx\, \left( \frac{\partial L}{\partial y}\eta +\frac{\partial L}{\partial y'}\eta' +\frac{\partial L}{\partial y''} \eta'' \right)\, . \end{align} We need to turn around the terms in $\eta'$ and $\eta''$. A first integration by parts will do this: \begin{align} \displaystyle\int_a^b\,dx\, \frac{\partial F}{\partial y'}\frac{d\eta}{dx} &=\frac{\partial F}{\partial y'}\eta(x)\Bigl\vert_a^b- \displaystyle\int_a^b\,dx\,\eta \frac{d}{dx}\left(\frac{\partial F}{\partial y'}\right)\, ,\\ \displaystyle\int_a^b\,dx\, \frac{\partial F}{\partial y''} \frac{d\eta'}{dx}&= \frac{\partial F}{\partial y''}\eta'(x) \Bigl\vert_a^b- \displaystyle\int_a^b\,dx\,\eta' \frac{d}{dx}\left(\frac{\partial F}{\partial y''}\right)\, . \end{align} We assume now that the function $\eta$ is chosen so that $\eta(b)=\eta(a)=0$ as before. In addition, we must also assume that $\eta'(b)=\eta'(a)=0$, a new condition.

In this way we have \begin{align} \displaystyle\int_a^b\,dx\, \frac{\partial L}{\partial y'}\eta'&= -\displaystyle\int_a^b\,dx\,\eta\, \frac{d}{dx} \left(\frac{\partial L}{\partial y'}\right)\, ,\\ \displaystyle\int_a^b\,dx\, \frac{\partial L}{\partial y''}\eta''&= -\displaystyle\int_a^b\,dx\,\eta'\, \frac{d}{dx}\left( \frac{\partial L}{\partial y''}\right)\, . \end{align}

We still need to turn $\eta'$ around one last time. Using integration by parts again: \begin{align} -\displaystyle\int_a^b\,dx\,\eta' \frac{d}{dx}\left(\frac{\partial F}{\partial y''}\right)= + \displaystyle\int_a^b\,dx\, \eta\,\frac{d^2}{dx^2}\left(\frac{\partial F}{\partial y''}\right) \end{align} where the boundary condition $\eta(b)=\eta(a)$ has been used to eliminate the boundary term.

Thus, putting all this together, we get \begin{align} 0=\frac{dI}{d\epsilon}\Bigl\vert_{\epsilon=0} =\displaystyle\int_a^b \,dx\,\eta\, \left(\frac{d^2}{dx^2}\left(\frac{\partial F}{\partial y''}\right)-\frac{d}{dx} \left(\frac{\partial F}{\partial y'}\right) +\frac{\partial F}{\partial y}\right)\, . \end{align} Since $\eta$ is arbitrary (up to the boundary conditions), we find therefore the function $L$ must satisfy the differential equation \begin{align} 0=\frac{d^2}{dx^2}\left( \frac{\partial L}{\partial y''}\right)- \frac{d}{dx}\left(\frac{\partial L}{\partial y'}\right)+\frac{\partial L}{\partial y}\, . \end{align} The generalization to $L$ containing yet more derivatives is obvious: for a the derivative of order $k$ we obtain a sign $(-1)^k$ as we need $k$ integrations by parts. Thus, we obtain the generalized Euler-Lagrange equation in the form \begin{align} 0&=\sum_{k}(-1)^k\frac{d^k}{dx^k} \left(\frac{\partial L}{\partial y^k}\right) \equiv E(L)\, ,\\ E&=\sum_k (-1)^k \frac{d^k}{dx^k} \frac{\partial }{\partial y^k}\, . \end{align}

There's some discussion of this (including how to properly define a conjugate momentum) in the following:

Riahi, F. "On Lagrangians with higher order derivatives." American Journal of Physics 40.3 (1972): 386-390.

and also in

Borneas, M. "On a generalization of the Lagrange Function." American Journal of Physics 27.4 (1959): 265-267.

Related Question